23
fiziks Institute for NET/JRF, GATE, IITJAM, JEST, TIFR and GRE in PHYSICAL SCIENCES Website: www.physicsbyfiziks.com Email: [email protected] 1 Head office fiziks, H.No. 23, G.F, Jia Sarai, Near IIT, Hauz Khas, New Delhi16 Phone: 01126865455/+919871145498 Branch office Anand Institute of Mathematics, 28B/6, Jia Sarai, Near IIT Hauz Khas, New Delhi16 JEST PHYSICS-2014 PART-A: 3 Mark Questions Q1. A dynamical system with two generalized coordinates 1 q and 2 q has Lagrangian 2 2 2 1 q q L + = . If 1 p and 2 p are the corresponding generalized momenta, the Hamiltonian is given by (a) ( ) 4 / 2 2 2 1 p p + (b) ( ) 4 / 2 2 2 1 q q + (c) ( ) 2 / 2 2 2 1 p p + (d) ( ) 4 / 2 2 1 1 q p q p + Ans.: (a) Solution: i i H qp L = L p q p q + = 2 2 1 1 2 2 1 1 1 1 1 p q q p q L = = = and 2 2 2 2 2 2 2 p q q p q L = = = 4 4 2 2 2 2 2 1 2 2 1 1 p p p p p p H + = ( ) 2 2 1 2 4 p p H + = Q2. In a certain inertial frame two light pulses are emitted, a distance 5 km apart and separated by s μ 5 . An observer who is traveling, parallel to the line joining the points where the pulses are emitted, at a velocity V with respect to this frame notes that the pulses are simultaneous. Therefore V is (a) c 7 . 0 (b) c 8 . 0 (c) c 3 . 0 (d) c 9 . 0 Ans.: (c) Solution: 3 6 2 1 2 1 5 10 , 5 10 sec x x mt t = × = × ( ) ( ) 2 2 1 1 2 1 2 1 2 2 2 2 1 2 2 2 2 2 2 1 1 1 v v v t x t x t t x x c c c t t v v v c c c + + = = 2 1 t t = 6 3 2 5 10 5 10 0 v c × × = 0.3 v c = Q3. Suppose a spin 2 / 1 particle is in the state ⎡+ = / 2 1 6 1 i υ

JEST 2014 Question n Solution

Embed Size (px)

DESCRIPTION

JEST 2014 Question n Solution

Citation preview

  • fiziks InstituteforNET/JRF,GATE,IITJAM,JEST,TIFRandGREinPHYSICALSCIENCES

    Website:www.physicsbyfiziks.comEmail:[email protected]

    Headofficefiziks,H.No.23,G.F,JiaSarai,NearIIT,HauzKhas,NewDelhi16Phone:01126865455/+919871145498

    BranchofficeAnandInstituteofMathematics,28B/6,JiaSarai,NearIITHauzKhas,NewDelhi16

    JEST PHYSICS-2014

    PART-A: 3 Mark Questions

    Q1. A dynamical system with two generalized coordinates 1q and 2q has Lagrangian 22

    21 qqL += . If

    1p and 2p are the corresponding generalized momenta, the Hamiltonian is given by

    (a) ( ) 4/2221 pp + (b) ( ) 4/2221 qq + (c) ( ) 2/2221 pp + (d) ( ) 4/2211 qpqp + Ans.: (a)

    Solution: i iH q p L= Lpqpq += 2211

    22 1111

    1

    pqqpqL === and 22

    2222

    2

    pqqpqL ===

    4422

    22

    21

    22

    11 ppppppH += ( )2 21 2

    4

    p pH

    + = Q2. In a certain inertial frame two light pulses are emitted, a distance 5 km apart and separated

    by s5 . An observer who is traveling, parallel to the line joining the points where the pulses are emitted, at a velocity V with respect to this frame notes that the pulses are simultaneous.

    Therefore V is

    (a) c7.0 (b) c8.0 (c) c3.0 (d) c9.0

    Ans.: (c)

    Solution: 3 62 1 2 15 10 , 5 10 secx x m t t = =

    ( ) ( )2 2 1 1 2 1 2 12 2 2

    2 1 2 2 2

    2 2 21 1 1

    v v vt x t x t t x xc c ct tv v vc c c

    + + = =

    2 1t t= 6 325 10 5 10 0vc = 0.3v c =

    Q3. Suppose a spin 2/1 particle is in the state

    +=/ 21

    61 i

  • fiziks InstituteforNET/JRF,GATE,IITJAM,JEST,TIFRandGREinPHYSICALSCIENCES

    Website:www.physicsbyfiziks.comEmail:[email protected]

    Headofficefiziks,H.No.23,G.F,JiaSarai,NearIIT,HauzKhas,NewDelhi16Phone:01126865455/+919871145498

    BranchofficeAnandInstituteofMathematics,28B/6,JiaSarai,NearIITHauzKhas,NewDelhi16

    If xS ( x component of the spin angular momentum operator) is measured what is the probability

    of getting 2/=+ ? (a) 3/1 (b) 3/2 (c) 6/5 (d) 6/1

    Ans.: (c)

    Solution: 0 11 02x

    S = = with eigenvalues

    2= and eigenvector corresponding to

    2= is 11

    12

    Now probability getting 2=+

    [ ]

    [ ]

    2

    211 1 11 1 1 222 6 512

    112 61 61 2 626

    ii

    pi

    i

    + + + = = = = +

    =

    Q4. For an optical fiber with core and cladding index of 45.11 =n and 44.12 =n , respectively, what is the approximate cut-off angle of incidence? Cut-off angle of incidence is defined as the

    incidence angle below which light will be guided.

    (a) o7 (b) o22 (c) o5 (d) o0

    Ans.: (a)

    Solution:

    2/12

    1

    21 1sin

    = nn where 45.1,44.1 12 == nn

    2/1

    1

    45.145.144.144.11sin

    = ( )11sin 0.11726 = 0 06.67 7 =

    Q5. A double pendulum consists of two equal masses m suspended by two strings of length l . What

    is the Lagrangian of this system for oscillations in a plane? Assume the angles 21 , made by the two strings are small (you can use 2/1cos 2 = ). Note: lg /0 = .

    (a)

    + 2220212022212 21

    21 mlL

  • fiziks InstituteforNET/JRF,GATE,IITJAM,JEST,TIFRandGREinPHYSICALSCIENCES

    Website:www.physicsbyfiziks.comEmail:[email protected]

    Headofficefiziks,H.No.23,G.F,JiaSarai,NearIIT,HauzKhas,NewDelhi16Phone:01126865455/+919871145498

    BranchofficeAnandInstituteofMathematics,28B/6,JiaSarai,NearIITHauzKhas,NewDelhi16

    (b)

    ++ 222021202122212 21

    21 mlL

    (c)

    + 222021202122212 21

    21 mlL

    (d)

    ++ 222021202122212 21

    21 mlL

    Ans.: (b)

    Solution: 11 sinlx = , 11 cosly = 2 1 2 2 1 2sin cosx x l y y l = + = + 2 1 2 2 1 2sin sin , cos cosx l l y l l = + = + 2 1 1 2 2 2 1 1 2 2cos cos , sin sinx l l y l l = + =

    21

    21

    22211

    2222

    22211

    2222

    22 sincoscos2coscos llllyx +++=+

    212122

    222

    2 sinsin2sin ll ++ ( )2 2 2 2 2 2 22 2 1 2 1 2 1 22 cosx y l l l + = + + also 2 2 2 21 1 1x y l + =

    VTL = ( ) 212222212121 mgymgyyxyxm +++= ( )( )2 2 2 2 2 2 21 1 2 1 2 1 2 1 21 2 cos 2 cos cos2L m l l l l mgl mgl = + + + + +

    2 22 2 2 1 2

    1 2 1 21 2 11 12 2 2 2 2

    g gL mll l

    = + + + + ( )1 2cos 1

    2 22 2 2 1 2

    1 2 1 212 2 2 2 2

    g g g gL mll l l l

    = + + + +

    comparing given options, option (b) is correct i.e.

    2 2

    2 2 2 20 11 2 1 2 0 2

    1 12 2 4

    L ml = + +

    1 l

    l

    m

    2

  • fiziks InstituteforNET/JRF,GATE,IITJAM,JEST,TIFRandGREinPHYSICALSCIENCES

    Website:www.physicsbyfiziks.comEmail:[email protected]

    Headofficefiziks,H.No.23,G.F,JiaSarai,NearIIT,HauzKhas,NewDelhi16Phone:01126865455/+919871145498

    BranchofficeAnandInstituteofMathematics,28B/6,JiaSarai,NearIITHauzKhas,NewDelhi16

    Q6. Circular discs of radius 1 m each are placed on a plane so as to form a closely packed triangular

    lattice. The number of discs per unit area is approximately equal to

    (a) 286.0 m (b) 243.0 m (c) 229.0 m (d) 214.0 m

    Ans.: (c)

    Solution: For closely packed hexagonal

    ,2ra = 1=r 1 1 16 2eff C f l

    n n n n= + +

    1 13 0 1 06 2eff

    n = + + 0.5effn =

    Occupancy ( )2 2effn r a rA= =

    ( )2

    2

    0.53 2

    4

    r

    r

    0.5 0.90643= =

    Now number of disc per unit area will be 29.0302.3

    9064.0 =

    Q7. What are the solutions to ( ) ( ) ( ) 02 =+ xfxfxf ? (a) xec x /1 (b) xcxc /21 + (c) 21 cxec x + (d) xx xecec 21 +

    Ans.: (d)

    Solution: Auxilary equation 0122 =+ DD ( )21 0D = 1, 1D = + + Roots are equal then ( ) ( )1 2 xf x c c x e= + ( ) 1 2x xf x c e c xe = +

    Q8. An ideal gas of non-relativistic fermions in 3-dimensions is at 0K. When both the number

    density and mass of the particles are doubled, then the energy per particle is multiplied by a

    factor

    (a) 2/12 (b) 1 (c) 3/12 (d) 3/12

    Ans.: (d)

    Solution: 32

    2

    43

    2

    = n

    mhEF at 0T K=

    Closely packed hexagonal

  • fiziks InstituteforNET/JRF,GATE,IITJAM,JEST,TIFRandGREinPHYSICALSCIENCES

    Website:www.physicsbyfiziks.comEmail:[email protected]

    Headofficefiziks,H.No.23,G.F,JiaSarai,NearIIT,HauzKhas,NewDelhi16Phone:01126865455/+919871145498

    BranchofficeAnandInstituteofMathematics,28B/6,JiaSarai,NearIITHauzKhas,NewDelhi16

    2n n = and 2m m =2 2

    22 23 333 3 12 2

    4 4 2 4 2Fh h nE nm m = =

    212 3

    33 22 4h nm

    =

    Q9. The value of 2.22.0 dxxex by using the one-segment trapezoidal rule is close to (a) 11.672 (b) 11.807 (c) 20.099 (d) 24.119

    Ans.: (c)

    Solution: 22.02.2 ==h ( ) ( )2.2 0.2 20.0992hI y y = + = xy xe=

    Q10. The Hamiltonian operator for a two-state system is given by

    ( )12212211 ++= H , where is a positive number with the dimension of energy. The energy eigenstates corresponding to the larger and smaller eigenvalues respectively are:

    (a) ( ) ( )2121,2121 ++ (b) ( ) ( )2121,2121 ++ (c) ( ) ( ) 2112,2121 ++ (d) ( ) ( ) 2112,2121 ++

    Ans.: (b)

    Solution: ( )12212211 ++= H ( )1 1 2H = + , ( )212 = H Lets check for option (b): ( ) ( )1 2 1 2 , 1 2 1 2+ + Now =H ( )1 2 1 2H + ( )1 2 1 2H H= + +

    ( )1 2 1 2H + ( ) ( )1 2 1 2H H + ( ) ( ) ( )1 2 2 1 1 2 + + ( )1 2 1 1 1 2 1 2 + + ( )22212 +

    ( )[ ]21212 + Now ( )2121 +H ( )[ ]2121 + H ( )2121 + HH

  • fiziks InstituteforNET/JRF,GATE,IITJAM,JEST,TIFRandGREinPHYSICALSCIENCES

    Website:www.physicsbyfiziks.comEmail:[email protected]

    Headofficefiziks,H.No.23,G.F,JiaSarai,NearIIT,HauzKhas,NewDelhi16Phone:01126865455/+919871145498

    BranchofficeAnandInstituteofMathematics,28B/6,JiaSarai,NearIITHauzKhas,NewDelhi16

    ( ) ( )( )1 2 2 1 1 2 + + ( ) ( )1 2 1 1 1 2 1 2 + + + ( ) 22212 ++ ( )[ ]22112 +

    Q11. Given the fundamental constants = (Plancks constant), G (universal gravitation constant) and c (speed of light), which of the following has dimension of length?

    (a) 3cG= (b) 5c

    G= (c) 3cG= (d)

    Gc8=

    Ans.: (a)

    Solution: [ ][ ] 21

    33

    23112

    TLTLMTML [ ] LL == 212

    2 1ML T = = , [ ]2312 == TLMmgrG Q12. Consider an eigenstate of 2L

    G and zL operator denoted by ml, . Let LnA

    G= denote an operator, where n is a unit vector parametrized in terms of two angles

    as ( ) ( ) cos,sin,cossin, =zyx nnn . The width A in ml, state is: (a) ( ) cos

    21 2 =mll + (b) ( ) sin

    21 2 =mll +

    (c) ( ) sin1 2=mll + (d) ( ) cos1 2=mll + Ans.: (b)

    Solution: A n L= G x y zx y zA L L Lr r r = + +

    sin sinsin cos cosx y z

    rr rA L L Lr r r

    = + + sin cos sin sin cosx y zA L L L = + +

    Now 22 AAA = sin cos sin sin cosx y zA L L L = + +

  • fiziks InstituteforNET/JRF,GATE,IITJAM,JEST,TIFRandGREinPHYSICALSCIENCES

    Website:www.physicsbyfiziks.comEmail:[email protected]

    Headofficefiziks,H.No.23,G.F,JiaSarai,NearIIT,HauzKhas,NewDelhi16Phone:01126865455/+919871145498

    BranchofficeAnandInstituteofMathematics,28B/6,JiaSarai,NearIITHauzKhas,NewDelhi16

    ( )cosA m = = 0 , 0x yL L= = 222222222 cossinsincossin zyx LLLA ++=

    ( ) ( )2 22 2 2 2 2 2 2 2 2 21 1sin cos sin sin cos2 2

    l l m l l mA m + + = + +

    = = =

    ( ) 22 2 2 2 2 2 2 21 sin sin cos cos2

    l l mA m + = + + = =

    ( ) 22 2 2 2 2 21 sin cos2

    l l mA m + = += =

    ( )( )222 2 2 2 2 2 2 2 21 sin cos cos2

    l l mA A A m m + = = + = = =

    ( ) 21sin

    2

    l l mA + = =

    Q13. The Laplace transformation of te t 4sin2 is

    (a) 254

    42 ++ ss (b) 204

    42 ++ ss

    (c) 204

    42 ++ ss

    s (d) 2042

    42 ++ ss

    s

    Ans.: (b)

    Solution: sina tL e bt ( )2 2b

    S a b= + +

    2 sin 4tL e t ( )2 24

    2 4S= + + 204

    42 ++= SS

    Q14. In the mixture of isotopes normally found on the earth at the present time, U23892 has an

    abundance of 99.3% and U23592 has an abundance of 0.7%. The measured lifetimes of these

    isotopes are 91052.6 years and 91002.1 years, respectively. Assuming that they were equally abundant when the earth was formed, the estimated age of the earth, in years is

  • fiziks InstituteforNET/JRF,GATE,IITJAM,JEST,TIFRandGREinPHYSICALSCIENCES

    Website:www.physicsbyfiziks.comEmail:[email protected]

    Headofficefiziks,H.No.23,G.F,JiaSarai,NearIIT,HauzKhas,NewDelhi16Phone:01126865455/+919871145498

    BranchofficeAnandInstituteofMathematics,28B/6,JiaSarai,NearIITHauzKhas,NewDelhi16

    (a) 9100.6 (b) 9100.1 (c) 8100.6 (d) 8100.1 Ans.: (a)

    Solution: If the number of 92 238U nuclei originally formed is N , the number present now is

    / / 6.52238t T tN Ne Ne = =

    where t is elapsed time in units of 910 year and T is life time of U . Since the number of 92 235U

    nuclei originally formed is. The number now present is

    /1.02235tN Ne=

    The present abundance of 92 235U is

    /1.02

    3 0.827235 235/ 6.52 3

    238 235 238

    1 4.967 10 143 6.07 10 0.827

    tt

    t

    N N Ne e tN N N Ne

    = = = = = = =+

    That is, the elapsed time is 96.0 10t = yr. Q15. Which of the following circuits will act like a 4-input NAND gate?

    (a) (b)

    (c) (d)

    Ans.: (d)

    Q16. Consider a three-state system with energies EE, and gE 3 (where g is a constant) and respective eigenstates

    =/01

    1

    21

    1

    =/

    211

    61

    2

    =/

    111

    31

    3

    If the system is initially (at 0=t ), in state

    =/

    001

    i

  • fiziks InstituteforNET/JRF,GATE,IITJAM,JEST,TIFRandGREinPHYSICALSCIENCES

    Website:www.physicsbyfiziks.comEmail:[email protected]

    Headofficefiziks,H.No.23,G.F,JiaSarai,NearIIT,HauzKhas,NewDelhi16Phone:01126865455/+919871145498

    BranchofficeAnandInstituteofMathematics,28B/6,JiaSarai,NearIITHauzKhas,NewDelhi16

    what is the probability that at a later time t system will be in state

    =/

    100

    f

    (a) 0 (b)

    =2

    3sin94 2 gt

    (c)

    =2

    3cos94 2 gt (d)

    =23sin

    94 2 gtE

    Ans.: (b)

    Q17. A hydrogen atom in its ground state is collided with an electron of kinetic energy 13.377 eV. The

    maximum factor by which the radius of the atom would increase is

    (a) 7 (b) 8 (c) 49 (d) 64

    Ans.: (c)

    Solution: 213.6

    nE eVn=

    1 13.6E eV = , 2 3.4E eV= , 3 1.5E eV= , 4 0.85E eV= , 5 0.54E eV= 6 0.3777E eV= , 7 0.2775E eV=

    Since Electron have kinetic energy 13.377 13.6 0.2775 7eV eV n= + = 2

    0nr a n= 049nr a =

    Q18. The formula for normal strain in a longitudinal bar is given by ,AEF= where F is normal

    force applied, A is cross-sectional area of the bar and E is Youngs modulus. If 2002.02.0,5.050 mANF == and 99 10110210 =E Pa, the maximum error in the

    measurement of strain is

    (a) 12100.1 (b) 111095.2 (c) 91022.1 (d) 91019.1 Ans.: (b)

    Solution: AEF=

  • fiziks InstituteforNET/JRF,GATE,IITJAM,JEST,TIFRandGREinPHYSICALSCIENCES

    Website:www.physicsbyfiziks.comEmail:[email protected]

    Headofficefiziks,H.No.23,G.F,JiaSarai,NearIIT,HauzKhas,NewDelhi16Phone:01126865455/+919871145498

    BranchofficeAnandInstituteofMathematics,28B/6,JiaSarai,NearIITHauzKhas,NewDelhi16

    9

    9

    10210101

    2.0002.

    505.0

    ++=++=

    EE

    AA

    FF

    .02476 0.2476 = = 11

    9

    0.2476 50 2.95 100.2 210 10

    = =

    Q19. A monoatomic gas consists of atoms with two internal energy levels, ground state 00 =E and an excited state EE =1 . The specific heat of the gas is given by

    (a) k23 (b) ( )2/2

    /2

    1 kTEkTE

    ekTeE+

    (c) ( )2/2/2

    123

    kTE

    kTE

    ekTeEk ++ (d) ( )2/2

    /2

    123

    kTE

    kTE

    ekTeEk +

    Ans.: (c)

    Solution: 0 10,E E E= = iEz e = 0 Ez e e = +

    ( )11lnln Eez += ( ) ( ) ( )

    1ln ln 11

    E EE

    U E z e E ee

    = = = + = +

    1

    E

    BEEeU k Te

    = =+

    2 2

    2

    1 . .

    1

    B B B B

    B

    E E E Ek T k T k T k T

    B BV E

    vk T

    E Ee E e Ee ek T k TU C

    Te

    + = = +

    2 22 2 2

    2 2 2 2 2

    2 2 2

    2 21 1 1

    B B B

    B B

    B B B

    E E EE Ek T k T k Tk T k T

    B B BV E E E

    k T k T k TB B

    E E Ee e ek T k T k T E e E eC

    e k T e k T e

    + = = = + + +

  • fiziks InstituteforNET/JRF,GATE,IITJAM,JEST,TIFRandGREinPHYSICALSCIENCES

    Website:www.physicsbyfiziks.comEmail:[email protected]

    Headofficefiziks,H.No.23,G.F,JiaSarai,NearIIT,HauzKhas,NewDelhi16Phone:01126865455/+919871145498

    BranchofficeAnandInstituteofMathematics,28B/6,JiaSarai,NearIITHauzKhas,NewDelhi16

    If gas will classically allowed then 32V B

    C k=

    and due to quantum mechanically 2

    2

    2 1

    B

    B

    Ek T

    V Ek T

    B

    E eC

    k T e

    = +

    ( )2 /

    22 /

    32 1

    E kT

    V B E kT

    E eC kkT e

    = ++

    Q20. Two large nonconducting sheets one with a fixed uniform positive charge and another with a

    fixed uniform negative charge are placed at a distance of 1 meter from each other. The

    magnitude of the surface charge densities are 2/8.6 mC =+ for the positively charged sheet and 2/3.4 mC = for the negatively charged sheet. What is the electric field in the region between the sheets?

    (a) CN /1030.6 5 (b) CN /1084.3 5 (c) CN /1040.1 5 (d) CN /1016.1 5

    Ans.: (a)

    Solution: Electric field between the sheet is 6 6

    0 0 0 0

    6.8 10 4.3 102 2 2 2 + = + = +

    66 5

    12

    11.2 10 0.626 10 6.3 10 /2 8.86 10

    N C

    =

    Q21. A monochromatic wave propagates in a direction making an angle o60 with the x -axis in the

    reference frame of source. The source moves at speed 54cv = towards the observer. The

    direction of the (cosine of angle) wave as seen by the observer is

    (a) 1413cos = (b)

    143cos = (c)

    613cos = (d)

    21cos =

    Ans.: (a)

  • fiziks InstituteforNET/JRF,GATE,IITJAM,JEST,TIFRandGREinPHYSICALSCIENCES

    Website:www.physicsbyfiziks.comEmail:[email protected]

    Headofficefiziks,H.No.23,G.F,JiaSarai,NearIIT,HauzKhas,NewDelhi16Phone:01126865455/+919871145498

    BranchofficeAnandInstituteofMathematics,28B/6,JiaSarai,NearIITHauzKhas,NewDelhi16

    Solution: 45cv = , ocos 60

    2xcu c = = , o 3sin 60

    2yu c c = =

    Now

    2

    42 5

    412 5

    x

    c cu

    c cc

    +=

    + 13

    14c= 13cos

    14 =

    Q22. A system of two circular co-axial coils carrying equal currents I along same direction having

    equal radius R and separated by a distance R (as shown in the figure below). The magnitude of

    magnetic field at the midpoint P is given by

    (a) RI

    220 (b)

    RI

    554 0

    (c) RI

    558 0 (d) 0

    Ans.: (c)

    Solution: ( )2

    03

    2 2 22

    IRBR d

    =+

    2

    01 3

    2 222

    4

    IRB

    RR

    = +

    , 2

    02 3

    2 222

    4

    IRB

    RR

    = +

    2Rd =

    21 BBB += 23

    0

    452

    2

    =

    R

    I32

    0 032

    4 85 55

    I IB

    RR

    = =

    Q23. Find the resonance frequency (rad/sec) of the circuit shown in the figure below

    (a) 1.41 (b) 1.0 (c) 2.0 (d) 1.73

    Ans.: (b)

    Solution: 2

    2

    1 1.0RLC L

    = = (where 2 , 2 , 0.25R L H C F= = = )

    V F25.03

    2

    H2

    RP

    i i

    X

    R

    Y

  • fiziks InstituteforNET/JRF,GATE,IITJAM,JEST,TIFRandGREinPHYSICALSCIENCES

    Website:www.physicsbyfiziks.comEmail:[email protected]

    Headofficefiziks,H.No.23,G.F,JiaSarai,NearIIT,HauzKhas,NewDelhi16Phone:01126865455/+919871145498

    BranchofficeAnandInstituteofMathematics,28B/6,JiaSarai,NearIITHauzKhas,NewDelhi16

    Q24. The temperature of a thin bulb filament (assuming that the resistance of the filament is nearly

    constant) of radius r and length L is proportional to

    (a) 2/14/1 Lr (b) rL2 (c) 14/1 rL (d) 12 Lr

    Ans.: (a)

    Q25. Ice of density 1 melts at pressure P and absolute temperature T to form water of density 2 . The latent heat of melting of 1 gram of ice is L . What is the change in the internal energy U resulting from the melting of 1 gram of ice?

    (a)

    +

    12

    11PL (b)

    12

    11PL

    (c)

    21

    11PL (d)

    +

    21

    11PL

    Ans.: (c)

    Solution: dU dQ W dQ pdV= =

    dU mL pdV= 2

    1

    2

    1dU L P d

    = 1 21 1L P

    =

    2

    1 1V dV d = =

    PART B: 1 MARK QUESTIONS

    Q26. A spherical air bubble is embedded in a glass slab. It will behave like a

    (a) Cylindrical lens (b) Achromatic lens (c) Converging lens (d) Diverging lens

    Ans.: (c)

    Q27. The acceleration experienced by the bob of a simple pendulum is

    (a) maximum at the extreme positions

    (b) maximum at the lowest (central) positions

    (c) maximum at a point between the above two positions

    (d) same at all positions

  • fiziks InstituteforNET/JRF,GATE,IITJAM,JEST,TIFRandGREinPHYSICALSCIENCES

    Website:www.physicsbyfiziks.comEmail:[email protected]

    Headofficefiziks,H.No.23,G.F,JiaSarai,NearIIT,HauzKhas,NewDelhi16Phone:01126865455/+919871145498

    BranchofficeAnandInstituteofMathematics,28B/6,JiaSarai,NearIITHauzKhas,NewDelhi16

    Ans.: (a)

    Solution: maT =sin , mgT =cos tanga = at o90=

    a is maximum at extreme position.

    Q28. A 100 ohms resistor carrying current of 1 Amp is maintained at a constant temperature of Co30

    by a heat bath. What is the rate of entropy increase of the resistor?

    (a) 3.3 Joules/K/sec (b) 6.6 Joules/K/sec

    (c) 0.33 Joules/K/sec (d) None of the above

    Ans.: (c)

    Solution: .qV = W i t R = 2W i Rt=

    now2 1 100 0.33

    30 273W i RtT T

    = = = +

    Q29. Consider a Hamiltonian system with a potential energy function given by ( ) 42 xxxV = . Which of the following is correct?

    (a) The system has one stable point (b) The system has two stable points

    (c) The system has three stable points (d) The system has four stable points

    Ans.: (a)

    Solution: ( ) 42 xxxV = 042 3 ==

    xxxV [ ] 0212 2 = xx

    0,2

    1=x 2 2

    22 2

    12

    12 12 2 12 4 02x

    V Vxdx dx =

    = = = <

    For stable point 0=xV and 0

    2

    >xV

    l T cosT

    mgsinT

  • fiziks InstituteforNET/JRF,GATE,IITJAM,JEST,TIFRandGREinPHYSICALSCIENCES

    Website:www.physicsbyfiziks.comEmail:[email protected]

    Headofficefiziks,H.No.23,G.F,JiaSarai,NearIIT,HauzKhas,NewDelhi16Phone:01126865455/+919871145498

    BranchofficeAnandInstituteofMathematics,28B/6,JiaSarai,NearIITHauzKhas,NewDelhi16

    020

    2

    2

    >=

    =xxV

    Q30. The lowest quantum mechanical energy of a particle confined in a one-dimensional box of size

    L is 2 eV. The energy of the quantum mechanical ground state for a system of three non-

    interacting spin 21 particles is

    (a) 6 eV (b) 10 eV (c) 12 eV (d) 16 eV

    Ans.: (c)

    Solution: 2 2

    1 2 22E eV

    ml= == , 2 14 8E E eV= =

    spin is 21

    then degeneracy 12 1 2 1 22

    S + = + = ground state 2 2 1 8 12eV eV eV + =

    Q31. The value of elastic constant for copper is about 1100 Nm and the atomic spacing is nm256.0 .

    What is the amplitude of the vibration of the Cu atoms at 300 K as a percentage of the

    equilibrium separation?

    (a) 4.55 % (b) 3.55 % (c) 2.55 % (d) 1.55 %

    Ans.: (b)

    Solution: we know that

    212

    E A= ma 910256.0 = for one dimension

    1.2 B

    K E K T= and KTEP21. =

    E kT= now 212B

    K T A= 2 Bk TA =

  • fiziks InstituteforNET/JRF,GATE,IITJAM,JEST,TIFRandGREinPHYSICALSCIENCES

    Website:www.physicsbyfiziks.comEmail:[email protected]

    Headofficefiziks,H.No.23,G.F,JiaSarai,NearIIT,HauzKhas,NewDelhi16Phone:01126865455/+919871145498

    BranchofficeAnandInstituteofMathematics,28B/6,JiaSarai,NearIITHauzKhas,NewDelhi16

    TkA

    2=23

    232 1.30 10 300 8.28 10100

    A

    = = 129.09 10 0.0090 nm= =

    Now x of 0.256 0.009nm nm= 0.00909 0.03551 3.5%0.256

    nmxnm

    = = = Q32. What is the contribution of the conduction electrons in the molar entropy of a metal with

    electronic coefficient of specific heat?

    (a) T (b) 2T (c) 3T (d) 4T Ans.: (a)

    Solution: 3VC BT AT= + Q33. Consider a system of 2N non-interacting spin 2/1 particles each fixed in position and carrying a

    magnetic moment . The system is immersed in a uniform magnetic field B. The number of spin up particle for which the entropy of the system will be maximum is

    (a) 0 (b) N (c) N2 (d) 2/N

    Ans.: (b)

    Solution: Let us consider n number of spin out of N2 particle have spin up remaining nN 2 is down. Number of ways nCN2= for spin 2

    1 (up)

    22

    N nCN = for spin 2

    1 down

    entropy lnkS = 2 22ln lnN NN n nS k C k C = +

    ( )( ) ( )( )2 ! 2 !ln ln

    ! 2 ! ! 2 !N NS K

    n N n n N n = +

    ( )( )[ ]!2ln!ln!2ln2 nNnNkS = ( ) ( ) ( ){ }2 2 ln 2 2 ln 2 ln 2 2S K N N N n n n N n N n N n = + !ln!ln NNNN =

    ( ) ( ) ( )2 2 ln 2 2 ln 2 ln 2 ln 2 2S K N N N n n n N N n n N n N n = + + +

  • fiziks InstituteforNET/JRF,GATE,IITJAM,JEST,TIFRandGREinPHYSICALSCIENCES

    Website:www.physicsbyfiziks.comEmail:[email protected]

    Headofficefiziks,H.No.23,G.F,JiaSarai,NearIIT,HauzKhas,NewDelhi16Phone:01126865455/+919871145498

    BranchofficeAnandInstituteofMathematics,28B/6,JiaSarai,NearIITHauzKhas,NewDelhi16

    ( ) ( )2 2 ln 2 ln 2 ln 2 ln 2S K N N n n N N n n N n = + now for entropy maximum at equilibrium for spin

    21 up particle

    0=dndS

    ( ) ( ) ( )22 1 ln 1 1 ln 22 2

    n N nK n N nn N n N n

    = + +

    ( )

    ++= nNnNn

    nNNnK 2ln

    222ln12

    ( )

    ++= nnNnNnNK ln2ln

    2212

    ( ) 02ln112 =

    ++=n

    nNk

    02 k 02ln =

    nnN 12 =

    nnN nN 22 = n N =

    Q34. When two different solids are brought in contact with each other, which one of the following is

    true?

    (a) Their Fermi energies become equal

    (b) Their band gaps become equal

    (c) Their chemical potentials become equal

    (d) Their work functions become equal

    Ans.: (c)

    Q35. For which gas the ratio of specific heats ( )vp CC / will be the largest? (a) mono-atomic (b) di-atomic (c) tri-atomic (d) hexa-atomic

    Ans.: (a)

    Solution: 21PV

    CC f

    = = + where f is degree of freedom.

  • fiziks InstituteforNET/JRF,GATE,IITJAM,JEST,TIFRandGREinPHYSICALSCIENCES

    Website:www.physicsbyfiziks.comEmail:[email protected]

    Headofficefiziks,H.No.23,G.F,JiaSarai,NearIIT,HauzKhas,NewDelhi16Phone:01126865455/+919871145498

    BranchofficeAnandInstituteofMathematics,28B/6,JiaSarai,NearIITHauzKhas,NewDelhi16

    For monoatomic: 3f = , For diatomic: 6f = , For Triatomic: 9f = For hexaatomic: 18f =

    Q36. A ball bounces off earth. You are asked to solve this quantum mechanically assuming the earth is

    an infinitely hard sphere. Consider surface of earth as the origin implying ( ) =0V and a linear potential elsewhere (i.e. ( ) mgxxV = for 0>x ). Which of the following wave functions is physically admissible for this problem (with 0>k ): (a) xe kx /=/ (b) 2kxxe=/ (c) kxAxe=/ (d) 2kxAe=/

    Ans.: (b)

    Solution: 2kxxe=

    For given potential, at 0,x = and x = wave function must vanish. Q37. Which functional form of potential best describes the interaction between a neutral atom and an

    ion at large distances (i.e. much larger than their diameters)

    (a) 2/1 rV (b) rV /1 (c) reV ar // (d) 3/1 rV Ans.: (a)

    Q38. An electron is executing simple harmonic motion along the y-axis in right handed coordinate

    system. Which of the following statements is true for emitted radiation?

    (a) The radiation will be most intense in xz plane

    (b) The radiation will be most intense in xy plane

    (c) The radiation will violate causality

    (d) The electrons rest mass energy will reduce due to radiation loss

    Ans.: (a)

    Solution: Oscillating electron does not emit radiation in the direction of oscillation.

    In the perpendicular direction of oscillation intensity is maximum.

    So in this case the intensity will be maximum along x and z - axis or xz - plane (intensity is also

    en xy -plane but less).

  • fiziks InstituteforNET/JRF,GATE,IITJAM,JEST,TIFRandGREinPHYSICALSCIENCES

    Website:www.physicsbyfiziks.comEmail:[email protected]

    Headofficefiziks,H.No.23,G.F,JiaSarai,NearIIT,HauzKhas,NewDelhi16Phone:01126865455/+919871145498

    BranchofficeAnandInstituteofMathematics,28B/6,JiaSarai,NearIITHauzKhas,NewDelhi16

    Q39. Two point objects A and B have masses 1000 Kg and 3000 Kg respectively. They are initially at

    rest with a separation equal to 1 m. Their mutual gravitational attraction then draws them

    together. How far from As original position will they collide?

    (a) 1/3 m (b) 1/2 m (c) 2/3 m (d) 3/4 m

    Ans.: (d)

    Solution: Since gravitational force is conservative, therefore they collide at their centre of mass

    ( ) 21 1 mxxm = ( )

    4313 == xxx

    Q40. If a proton were ten times lighter, then the ground state energy of the electron in a hydrogen

    atom would have been

    (a) Less (b) More

    (c) The same (d) Depends on the electron mass

    Ans.: (b)

    Solution: 526.1300545.16.136.13

    22 ===e

    e

    en m

    mnmn

    E

    1.0054510

    p e pp e

    e p

    M m mM m

    m m = = =+

    Q41. Let us write down the Lagrangian of a system as ( ) xcxkxxmxxxxL ++= 2,, . What is the dimension of c ?

    (a) 3MLT (b) 2MT (c) MT (d) 12 TML

    Ans.: (c)

    Solution: According to dimension rule same dimension will be added or subtracted then dimension of

    =xMx dimension of Cxx [ ] [ ]2 2 3ML T C L LT = [ ] [ ][ ] [ ]MTML TMLC ==

    32

    22

    1mA

    x x1 2mB

    m1

  • fiziks InstituteforNET/JRF,GATE,IITJAM,JEST,TIFRandGREinPHYSICALSCIENCES

    Website:www.physicsbyfiziks.comEmail:[email protected]

    Headofficefiziks,H.No.23,G.F,JiaSarai,NearIIT,HauzKhas,NewDelhi16Phone:01126865455/+919871145498

    BranchofficeAnandInstituteofMathematics,28B/6,JiaSarai,NearIITHauzKhas,NewDelhi16

    Q42. The Dirac delta function ( )x satisfies the relation ( ) ( ) ( ) = 0fdxxxf for a well behaved function ( )xf . If x has the dimension of momentum then (a) ( )x has the dimension of momentum (b) ( )x has the dimension of ( )2momentum (c) ( )x is dimensionless (d) ( )x has the dimension of ( ) 1momentum

    Ans.: (d)

    Solution: ( ) ( ) ( ) = 0fdxxxf ( ) ( ) ( ) ( )[ ] ( ) ( )[ ] ( ) [ ]100 === PxfPxxffdxxxf Since, ( )[ ] ( )[ ]0fxf = If ( ) += xxF is force [ ]2TLM ( ) =0F is also [ ]2TLM

    Q43. The operator A and B share all the eigenstates. Then the least possible value of the product of

    uncertainties BA is (a) = (b) 0 (c) 2/= (d) Determinant (AB)

    Ans.: (b)

    Solution: [ ]2ABBA

    0 BA A and B have share their eigen values so [ ] 0=AB

    Q44. The resolving power of a grating spectrograph can be improved by

    (a) recording the spectrum in the lowest order

    (b) using a grating with longer grating period

    (c) masking a part of the grating surface

    (d) illuminating the grating to the maximum possible extent

  • fiziks InstituteforNET/JRF,GATE,IITJAM,JEST,TIFRandGREinPHYSICALSCIENCES

    Website:www.physicsbyfiziks.comEmail:[email protected]

    Headofficefiziks,H.No.23,G.F,JiaSarai,NearIIT,HauzKhas,NewDelhi16Phone:01126865455/+919871145498

    BranchofficeAnandInstituteofMathematics,28B/6,JiaSarai,NearIITHauzKhas,NewDelhi16

    Ans.: (d)

    Solution: R P nN = = where N - Number of slit and n - order of diffraction.

    Q45. The value of limit

    11lim 6

    10

    ++

    zz

    iz

    is equal to

    (a) 1 (b) 0 (c) -10/3 (d) 5/3

    Ans.: (d)

    Solution: 11lim 6

    10

    ++

    zz

    iz 35

    610

    610lim

    610lim

    4

    5

    9

    = z

    zz

    iziz

    Q46. A conducting sphere of radius r has charge Q on its surface. If the charge on the sphere is

    doubled and its radius is halved, the energy associated with the electric field will

    (a) increase four times (b) increase eight times

    (c) remain the same (d) decrease four times

    Ans.: (b)

    Solution: 2

    2 2 2 20 01 22 0

    0 0

    4 44 2 2 8

    R

    R

    Q QE rW E r dr E r dr Wr R

    = = + =

    ( )2 2

    00

    2 8 888

    2

    Q QW WR R = = =

    Q47. Three sinusoidal waves have the same frequency with amplitude 2/, AA and 3/A while their

    phase angles are 2/,0 and respectively. The amplitude of the resultant wave is (a)

    611A (b)

    32A (c)

    65A (d)

    67A

    Ans.: (c)

    Solution: ( ) ( ) +=

    +=+= tAytAytAy sin3

    ,2

    sin2

    ,0sin 321

  • fiziks InstituteforNET/JRF,GATE,IITJAM,JEST,TIFRandGREinPHYSICALSCIENCES

    Website:www.physicsbyfiziks.comEmail:[email protected]

    Headofficefiziks,H.No.23,G.F,JiaSarai,NearIIT,HauzKhas,NewDelhi16Phone:01126865455/+919871145498

    BranchofficeAnandInstituteofMathematics,28B/6,JiaSarai,NearIITHauzKhas,NewDelhi16

    tAtAtAyyyy sin3

    cos2

    sin321 +=++=

    tAtA cos2

    sin3

    2 +

    6

    536

    2549

    423

    2 22222 AAAAAAA ==+=

    +

    =

    Q48. The value of integral

    = c dzz zI 2sin with c a circle 2=z , is (a) 0 (b) i2 (c) i (d) i

    Ans.: (c)

    Solution: = C z zI 2sin pole 202 == zz Residue at

    2=z 2=z so it will be lies within the contour

    ( ) =

    =

    C

    iz

    emg iRz

    eI 22

    2

    Res 22

    22

    2 2/

    2

    ie

    z

    ez iiz

    z

    ==

    =

    (taking imaginary part)

    Residue = 21

    Now iiI == 221

    Q49. Consider a square well of depth 0V and width a with aV0 fixed. Let 0V and 0a . This potential well has

  • fiziks InstituteforNET/JRF,GATE,IITJAM,JEST,TIFRandGREinPHYSICALSCIENCES

    Website:www.physicsbyfiziks.comEmail:[email protected]

    Headofficefiziks,H.No.23,G.F,JiaSarai,NearIIT,HauzKhas,NewDelhi16Phone:01126865455/+919871145498

    BranchofficeAnandInstituteofMathematics,28B/6,JiaSarai,NearIITHauzKhas,NewDelhi16

    (a) No bound states (b) 1 bound state

    (c) 2 bound states (d) Infinitely many bound states

    Ans.: (b)

    Solution: It forms delta potential so it has only one bound state.

    Q50. If hydrogen atom is bombarded by energetic electrons, it will emit

    (a) K X - rays (b) -rays (c) Neutrons (d) none of the above

    Ans.: (d)